Logo Studenta

ana1_1

¡Este material tiene más páginas!

Vista previa del material en texto

CAPÍTULO I.
EL CUERPO ORDENADO
DE LOS NÚMEROS
REALES
SECCIONES
A. Elementos notables en R.
B. Congruencias. Conjuntos numerables.
C. Método de inducción completa.
D. Desigualdades y valor absoluto.
E. Ejercicios propuestos.
1
A. ELEMENTOS NOTABLES EN R.
Sea S 6= ∅ un subconjunto del conjunto R de números reales.
a) Diremos que S está acotado superiormente por u, o que u es cota superior
de S, cuando x ≤ u, ∀x ∈ S.
Una cota superior u de S se llama supremo de S si ningún número
menor que u es cota superior de S, es decir cuando u es la menor de
las cotas superiores de S.
Si u es el supremo de S y u ∈ S, entonces u se llama máximo de S.
b) Análogamente a los anteriores se pueden definir los conceptos siguientes:
El conjunto S está acotado inferiormente por i, o bien i es cota inferior
de S, cuando x ≥ i, ∀x ∈ S.
Una cota inferior i de S se llama ı́nfimo de S si ningún número mayor
que i es cota inferior de S, es decir cuando i es la mayor de las cotas
inferiores de S.
Si i es el ı́nfimo de S e i ∈ S, entonces i se llama mı́nimo de S.
Una propiedad fundamental de los números reales la constituye el axioma
del supremo, el cual establece que todo subconjunto de R no vaćıo y acotado
superiormente posee supremo. De él se deduce otra condición análoga para
la existencia de ı́nfimo. Además estas propiedades son caracteŕısticas del
conjunto R.
PROBLEMA 1.1.
Hallar el supremo, ı́nfimo, máximo y mı́nimo (cuando existan) de
los siguientes conjuntos:
a) C =
{sennπ
n
| n ∈ N
}
.
b) D =
{cos nπ
n
| n ∈ N
}
.
2
Solución
a) Como sennπ = 0 si n ∈ N, entonces C = {0}, de modo que
supC = ı́nf C = máxC = mı́nC = 0.
b) Como cos nπ =
{
1 si n es par
−1 si n es impar,
entonces
D = {−1, 1/2,−1/3, 1/4, . . . } = {−1,−1/3,−1/5, . . . }∪{1/2, 1/4, 1/6, . . . }.
Luego ı́nf D = mı́nD = −1 y sup D = máxD = 1/2.
PROBLEMA 1.2.
Hallar el supremo y el ı́nfimo, cuando existan, de los siguientes
conjuntos de números reales, especificando cuáles tienen elemento
máximo o mı́nimo, es decir, cuándo el supremo o el ı́nfimo perte-
necen al conjunto.
a) A =
{
1
n
| n ∈ N
}
.
b) B =
{
1
n
| n ∈ Z, n 6= 0
}
.
c) C =
{
x | x = 0 ó x = 1
n
, n ∈ N
}
.
d) D =
{
x | 0 ≤ x <
√
2, x ∈ Q
}
.
e) E =
{
x | x2 + x + 1 ≥ 0
}
.
f) F =
{
x | x2 + x− 1 < 0
}
.
g) G =
{
x | x < 0, x2 + x− 1 < 0
}
.
Solución
a) Como A = {1, 1/2, 1/3, . . . }, es claro que
supA = máxA = 1 e ı́nf A = 0 6∈ A,
por lo que no existe mı́nA.
3
b) B = {1, 1/2, 1/3, . . . } ∪ {−1,−1/2,−1/3, . . . } por lo que
máx B = supB = 1 e ı́nf B = mı́nB = −1.
c) Podemos escribir C = A ∪ {0} y tenemos
máx C = supC = 1 e ı́nf C = mı́nC = 0.
d) Como supD =
√
2 6∈ D, no existe máx D. Sin embargo ı́nf D = mı́nD =
0.
e) Al resolver la ecuación x2 + x + 1 = 0 tenemos x =
−1±
√
1− 4
2
y las
ráıces son imaginarias, por lo que x2 + x + 1 ≥ 0, ∀x ∈ R, es decir
E = R que no está acotado ni superior ni inferiormente.
f) Las ráıces de la ecuación x2+x−1 = 0 son x = −1±
√
5
2
. Luego la inecua-
ción x2+x−1 < 0 sólo se verifica en el intervalo F =
(
−1−
√
5
2
,
−1 +
√
5
2
)
.
Para este intervalo tenemos
supF =
−1 +
√
5
2
6∈ F e ı́nf F = −1−
√
5
2
6∈ F,
por lo que no existen ni el máximo ni el mı́nimo de F .
g) Como G = F ∩ {x | x < 0} = {x | (−1−
√
5)/2 < x < 0}, resulta que
supG = 0 6∈ G e ı́nf G = −1−
√
5
2
6∈ G
y en este caso tampoco el conjunto posee máximo ni mı́nimo.
PROBLEMA 1.3.
Hallar el supremo, ı́nfimo, máximo y mı́nimo del conjunto
I =
{
1
n
+ (−1)n | n ∈ N
}
.
4
Solución
Descomponemos el conjunto como
I = {1 + 1/n | n es par} ∪ {−1 + 1/n | n es impar}
= {3/2, 5/4, 7/6, . . . } ∪ {0,−2/3,−4/5, . . . }.
De aqúı es fácil ver que
sup I = máx I = 3/2 e ı́nf I = −1 6∈ I,
por lo que no existe el mı́nimo de I.
PROBLEMA 1.4.
Calcular el supremo y el ı́nfimo de los conjuntos
A =
∞⋃
n=1
(
− 1
n
,
1
n
)
.
B =
∞⋂
n=1
(
− 1
n
,
1
n
)
.
C =
∞⋂
n=1
[
1
2n
,
1
2n− 1
]
.
Solución
a) Como cada intervalo (−1/n, 1/n) está contenido en el anterior, resulta
que
A =
∞⋃
n=1
(
− 1
n
,
1
n
)
= (−1, 1)
y supA = 1, ı́nf A = −1.
b) Como 0 es el único punto que pertenece a todos los intervalos anteriores,
tenemos que
B =
∞⋂
n=1
(
− 1
n
,
1
n
)
= {0}
con lo que supB = ı́nf B = 0.
5
c) Como ninguno de los intervalos [1/2n, 1/(2n− 1)] tiene puntos en común,
resulta que
C =
∞⋂
n=1
[
1
2n
,
1
2n− 1
]
= ∅
y no posee supremo ni ı́nfimo.
PROBLEMA 1.5.
Sean A y B dos conjuntos no vaćıos de números tales que x ≤
y, ∀x ∈ A, y ∈ B.
a) Demostrar que supA ≤ y, ∀y ∈ B y que x ≤ ı́nf B, ∀x ∈ A.
b) Demostrar que supA ≤ ı́nf B.
Solución
a) Sea a = supA. Por definición, i) a ≥ x, ∀x ∈ A (a es cota superior de
A), y
ii) si a′ ≥ x, ∀x ∈ A, entonces a ≤ a′ (a es la cota superior más
pequeña de A).
Por ii), como todo y ∈ B es cota superior de A, a = supA ≤ y.
Análogamente, si llamamos ı́nf B = b, también la definición indica que
i) b ≤ y, ∀y ∈ B y ii) si b′ ≤ y, ∀y ∈ B, entonces b ≥ b′.
Como todo x ∈ A verifica ii), tenemos que ı́nf B ≥ x.
b) Debido a que todo elemento de A es cota inferior de B, es claro que
supA ≤ ı́nf B puesto que ı́nf B es la máxima cota inferior.
PROBLEMA 1.6.
Sea S un subconjunto no vaćıo de R y denotamos por s a una cota
superior de S. Demostrar que
s = supS ⇐⇒ ∀ε > 0, ∃a ∈ S : s− ε < a ≤ s.
6
Solución
=⇒: Si s = supS, entonces ∀x ∈ S, s ≥ x y para todo ε > 0, s − ε
no es cota superior de S, porque s − ε < s. Esto quiere decir que existe
a ∈ S tal que a > s − ε. Pero también a ≤ s pues s = supS. En resumen,
∀ε > 0,∃a ∈ S : s− ε < a ≤ s.
⇐=: Si λ = supS, no puede ser λ < s pues eligiendo ε < s− λ, tendŕıamos
que λ < s − ε y no podŕıa existir a ∈ S tal que s − ε < a en contra de la
hipótesis. Luego ha de ser λ ≥ s y como supS es la mı́nima cota superior,
supS = s.
PROBLEMA 1.7.
a) Sean A y B dos conjuntos no vaćıos con A ⊂ B. Probar que supA ≤
supB e ı́nf A ≥ ı́nf B.
b) Sean A y B dos conjuntos acotados superiormente, y definimos
A + B = {x + y | x ∈ A, y ∈ B}. Demostrar que sup(A + B) =
supA + supB, ı́nf(A + B) = ı́nf A + ı́nf B.
c) Si A = {xi}i∈I , B = {yi}i∈I y C = {xi + yi}i∈I , demostrar que
supC ≤ supA + supB e ı́nf C ≥ ı́nf A + ı́nf B.
Solución
a) Si llamamos b = supB, entonces b ≥ y, ∀y ∈ B. Por tanto b ≥ y, ∀y ∈ A,
lo que quiere decir que b es cota superior de A. Como sup A es la
mı́nima cota superior, se deduce que supA ≤ supB.
Análogamente, ı́nf B es una cota inferior de A y como ı́nf A es la mayor
de todas ellas, resulta que ı́nf A ≥ ı́nf B.
b) Sean a = supA y b = supB. Entonces a ≥ x, ∀x ∈ A y b ≥ y, ∀y ∈ B.
Por tanto a + b ≥ x + y, ∀x ∈ A, y ∈ B y a + b es cota superior de
A + B. Probemos que es la mı́nima:
Como a = supA, por el problema anterior, ∀ε > 0, ∃a0 ∈ A : a−ε/2 <
a0 ≤ a, y como b = supB, ∀ε > 0, ∃b0 ∈ B : b− ε/2 < b0 ≤ b.
7
Sumando miembro a miembro deducimos que existe a0 + b0 ∈ A + B
tal que a+ b− ε < a0 + b0 ≤ a+ b, lo que indica que a+ b es la mı́nima
cota superior de A + B, es decir, sup(A + B) = a + b = supA + supB.
La prueba de que ı́nf(A + B) = ı́nf A + ı́nf B es análoga a la anterior,
con las modificaciones obvias.
c) Obsérvese que C = {xi + yi}i∈I es un subconjunto de A + B pues
A + B = {xi + yj | xi ∈ A, yj ∈ B}. Por los apartados a) y b) se
deduce que
supC ≤ sup(A+B) = supA+supB e ı́nf C ≥ ı́nf(A+B) = ı́nf A+ı́nf B.
PROBLEMA 1.8.
Probar que R es arquimediano, es decir que verifica la propiedad:
∀a, b ∈ R, a > 0, b ≥ 0,∃n ∈ N tal que na > b.
Solución
Si fuera falso, na ≤ b, ∀n ∈ N. Por tanto, n ≤ b/a, n = 1, 2, . . . lo que
quiere decir que N está acotado superiormente.
Por el axioma del supremo, debe existir k = sup N, lo que a su vez implica
que k − 1 no es cota superior. Entonces existe m ∈ N tal que k − 1 < m de
donde k < m + 1 y m + 1 ∈ N lo que es absurdo.
PROBLEMA 1.9.
Sean a, b ∈ R con a < b. Probar que existe r ∈ Q tal que a < r < b.
8
Solución
Si aplicamos la propiedad arquimediana de los números reales con1 y b− a
demostrada en el problema anterior, tenemos que existe n ∈ N tal que n(b−
a) > 1.
Por una parte, si b ≥ 0, existe p ∈ N tal que p · 1/n ≥ b; llamamos m al
menor de tales enteros. Entonces
m− 1
n
< b ≤ m
n
.
Si fuera
m− 1
n
≤ a, entonces b− a ≤ m
n
− m− 1
n
=
1
n
, lo que contradice la
elección de n. Por tanto
a <
m− 1
n
< b y
m− 1
n
∈ Q.
Por otra parte, si b ≤ 0, entonces −a > −b ≥ 0 y, por la primera parte,
existe r ∈ Q tal que −b < r < −a, es decir, a < −r < b y −r ∈ Q.
Observación. La propiedad anterior indica que, a pesar de que el conjunto
de números racionales es mucho menor que el de los reales, está uniforme-
mente distribuido entre estos, pues siempre hay algún racional comprendido
entre dos reales.
B. CONGRUENCIAS. CONJUNTOS NUMERABLES.
Recordamos el concepto de aplicación biyectiva entre conjuntos:
Una aplicación f : A → B entre dos conjuntos cualesquiera A y B es
inyectiva cuando dados dos elementos cualesquiera x1, x2 ∈ A, se verifica
que f(x1) = f(x2) =⇒ x1 = x2. Equivalente a la implicación anterior es
la contrarrećıproca, es decir f es inyectiva cuando x1 6= x2 =⇒ f(x1) 6=
f(x2).
Por otra parte, la aplicación f es sobre cuando ∀y ∈ B, ∃x ∈ A tal que
f(x) = y, es decir cuando todos los elementos de B son imagen de algún
elemento de A.
Si una aplicación es a la vez inyectiva y sobre, se dice que es biyectiva.
9
Cuando una aplicación f : A → B es biyectiva, existe, y también es biyectiva,
la llamada aplicación inversa f−1 : B → A definida por f−1(y) = x cuando
f(x) = y, para todo y ∈ B.
Decimos que dos conjuntos A y B son congruentes o equipotentes, y lo repre-
sentamos como A ∼ B, cuando existe una aplicación biyectiva f : A → B.
Es sencillo probar que la relación de congruencia aśı definida es de equiva-
lencia.
Si llamamos cardinal de un conjunto al número de elementos o puntos que
posee, la definición anterior permite decir que dos conjuntos congruentes
tienen el mismo cardinal.
Si clasificamos los conjuntos de números reales con respecto a su cardinal,
tenemos:
- Conjunto finito es aquél cuyo cardinal es finito (tiene sólo un número finito
de elementos).
- Conjunto infinito numerable es aquel conjunto congruente con el conjunto
de números naturales. Por ejemplo, el conjunto de los números racionales es
numerable.
- Conjunto infinito no numerable es el conjunto que no es congruente con N
ni con cualquier subconjunto de N.
Un intervalo como (0, 1) no es numerable y su cardinal, llamado potencia
del continuo, se suele expresar con la letra c.
PROBLEMA 1.10.
Sean los ćırculos concéntricos
C1 = {(x, y) | x2 + y2 = a2}, C2 = {(x, y) | x2 + y2 = b2}
con 0 < a < b. Establecer geométricamente una aplicación biyectiva
entre C1 y C2.
Solución
Definimos la aplicación f : C2 → C1 que a cada x ∈ C2 le hace corresponder
el punto f(x) ∈ C1 de intersección del radio que va del centro 0 al punto x
con la circunferencia C1, como se ilustra en la figura.
10
x
f(x)
O
C1
C2
Resulta que f es claramente inyectiva y sobre por lo que define una aplicación
biyectiva entre C1 y C2.
PROBLEMA 1.11.
Demostrar que dos segmentos arbitrarios tienen tantos puntos el
uno como el otro.
Solución
Dados los segmentos AB y A′B′, llamamos P al punto de intersección de
AB′ y BA′. A cada punto C de AB le hacemos corresponder el punto C ′ de
intersección de CP con A′B′ como indica la figura.
A C B
P
A′ C ′ B′
La aplicación C 7→ C ′ es biyectiva pues cada punto de AB tiene una imagen
única en A′B′ y cada punto de A′B′ es la imagen de un único punto de AB.
PROBLEMA 1.12.
Demostrar las siguientes congruencias entre conjuntos:
a) [0, 1] ∼ (0, 1).
b) [0, 1] ∼ [0, 1).
c) [0, 1] ∼ (0, 1].
11
Solución
a) Si escribimos [0, 1] = {0, 1, 1/2, 1/3, . . . }∪A y (0, 1) = {1/2, 1/3, . . . }∪A,
donde A = [0, 1]\{0, 1, 1/2, 1/3, . . . } = (0, 1)\{1/2, 1/3, . . . }, podemos
definir la función f : [0, 1] → (0, 1) por
f(x) =

2 si x = 0,
1/(n + 2) si x = 1/n, n ∈ N,
x si x 6= 0, 1/n, n ∈ N.
Esta función es biyectiva por lo que [0, 1] ∼ (0, 1).
b) La función f : [0, 1] → [0, 1) definida por
f(x) =
{
1/(n + 1) si x = 1/n, n ∈ N,
x si x 6= 1/n, n ∈ N.
es biyectiva por lo que [0, 1] ∼
[0, 1).
c) Sea f : [0, 1) → (0, 1] la función definida por f(x) = 1− x. Esta función
es biyectiva y por tanto [0, 1) ∼ (0, 1]. Como por b) [0, 1] ∼ [0, 1),
por la transitividad de la congruencia entre conjuntos se deduce que
[0, 1] ∼ (0, 1].
PROBLEMA 1.13.
Demostrar que cada uno de los intervalos [a, b], (a, b), [a, b), (a, b]
tiene la potencia del continuo, es decir tiene cardinal c.
Solución
La fórmula f(x) = a + (b− a)x define las funciones biyectivas
f : [0, 1] → [a, b], f : [0, 1) → [a, b), f : (0, 1) → (a, b) y f : (0, 1] → (a, b].
Como ya sabemos que [0, 1] ∼ [0, 1), [0, 1] ∼ (0, 1) y [0, 1] ∼ (0, 1], por la
transitividad se deduce que [0, 1] ∼ [a, b], [0, 1] ∼ (a, b), [0, 1] ∼ [a, b) y
[0, 1] ∼ (a, b].
12
PROBLEMA 1.14.
Demostrar que R tiene cardinal c.
Solución
Es fácil comprobar (ver gráfica adjunta) que la función f : (−1, 1) → R
definida por f(x) =
x
1− |x|
es biyectiva por lo que (−1, 1) ∼ R.
Como ya sabemos que (0, 1) ∼ (−1, 1), por la transitividad se deduce que
(0, 1) ∼ R, luego card (R) = c.
PROBLEMA 1.15.
Sea E = {2, 4, 6, . . . } el conjunto de los números pares. ¿Es biyec-
tiva la aplicación f : N → E definida por f(x) = 2x? ¿Es N ∼ E?
Solución
La función f es inyectiva porque si f(x) = f(x′), entonces 2x = 2x′, de
donde x = x′.
13
También es sobre porque dado cualquier y ∈ E, existe x ∈ N tal que y = 2x,
es decir, y = f(x).
Como f es biyectiva, N ∼ E.
PROBLEMA 1.16.
¿Es numerable la sucesión {a1, a2, . . . } con ai 6= aj si i 6= j?
Solución
Debido a que toda sucesión es una aplicación de N en el conjunto de sus
elementos, f(n) = an, si los an son distintos, la función es biyectiva y la
sucesión forma un conjunto numerable.
PROBLEMA 1.17.
Demostrar que A × B ∼ B × A, (A × B) × C ∼ A × (B × C), para
cualesquiera conjuntos A,B, C.
Solución
Para la demostración basta considerar las aplicaciones biyectivas siguien-
tes:
f : A×B → B ×A definida por f(a, b) = (b, a), a ∈ A, b ∈ B;
f : (A×B)×C → A× (B×C) por f((a, b), c) = (a, (b, c)), a ∈ A, b ∈ B, c ∈
C.
PROBLEMA 1.18.
Demostrar que N× N es numerable.
14
Solución
El conjunto N×N puede escribirse como una sucesión de elementos distintos
aśı: (1, 1), (2, 1), (1, 2), (1, 3), (2, 2), (3, 1), . . . según se ilustra en el diagrama.
4
3
2
1
0 1 2 3 4
Según el problema 1.16, se puede definir una biyección entre N × N y N.
PROBLEMA 1.19.
Demostrar que M ×M es numerable, donde M = N ∪ {0}.
Solución
Como todo entero positivo a ∈ N puede escribirse de una sola manera en la
forma a = 2r(2s + 1) con r, s ∈ M , la función f : N → M ×M definida por
f(a) = (r, s) es biyectiva. Luego M ×M es numerable.
PROBLEMA 1.20.
Probar que Z es numerable.
15
Solución
La función f : N → Z definida por f(n) =

0 si n = 1,
n′ si n = 2n′, n′ ∈ N,
−n′ si n = 2n′ + 1, n′ ∈ N,
(que también se puede expresar como f(n) =
{
−n/2 + 1/2 si n es impar
n/2 si n es par
)
es evidentemente biyectiva, lo que muestra que Z es numerable.
PROBLEMA 1.21.
Demostrar que Q es numerable.
Solución
Llamamos Q+ al conjunto de los números racionales positivos y Q− al con-
junto de los números racionales negativos. Entonces Q = Q+∪{0}∪Q−.
Sea f : Q+ → N × N definido por f(p/q) = (p, q), donde expresamos el
número racional p/q como cociente de dos enteros positivos primos entre śı.
Es inmediato comprobar que f es inyectiva y que, por tanto, Q+ es equipo-
tente a un subconjunto de N×N. Como este último conjunto es numerable,
también lo es Q+.
Se comprueba asimismo que Q− es numerable (basta definir f(−p/q) =
(p, q)). Como Q es unión de dos conjuntos numerables y uno finito, Q es
numerable.
PROBLEMA 1.22.
Demostrar que el conjunto P de todos los polinomios p(x) = a0 +
a1x + · · ·+ amxm con coeficientes enteros es numerable.16
Solución
Para todo par de números naturales (m,n) sea P(m,n) el conjunto de los poli-
nomios p(x) = a0+a1x+· · ·+amxm de grado m para los que |a0|+|a1|+· · ·+
|am| = n. Es claro que P(m,n) es finito. Como además P =
⋃
(m,n)∈N×N
P(m,n) es
unión numerable de conjuntos finitos, se deduce que P es numerable.
PROBLEMA 1.23.
Un número real r se llama algebraico si es solución de una ecua-
ción polinómica p(x) = a0 + a1x + · · · + anxn = 0 con coeficientes
enteros. Demostrar que el conjunto de los números algebraicos es
numerable.
Solución
Es evidente, según el problema anterior, que el conjunto de tales ecuaciones
polinómicas E = {p1(x) = 0, p2(x), p3(x) = 0, . . . } es numerable. Si llama-
mos Ai = {x | x es solución de pi(x) = 0}, como todo polinomio de grado n
no puede tener más de n ráıces, todo Ai es finito. Por consiguiente, y como
A =
⋃
i∈N
Ai es unión numerable de conjuntos finitos, A es numerable.
PROBLEMA 1.24.
Un número real se llama trascendente si no es algebraico (por
ejemplo, π y e son trascendentes). Demostrar que el conjunto de
los números trascendentes no es numerable.
Solución
Si A es el conjunto de los números algebraicos, R \ A es el conjunto de los
números trascendentes. Como R = A ∪ (R \ A), al ser A numerable y R
17
no numerable, R \ A no puede ser numerable (en caso contrario, R seŕıa
numerable por ser unión de dos conjuntos numerables).
PROBLEMA 1.25.
Demostrar que un subconjunto de un conjunto numerable es finito
o numerable.
Solución
Sea A = {a1, a2, . . . } un conjunto numerable y sea B ⊂ A.
Si B = ∅, entonces B es finito.
Si B 6= ∅, sea an1 el primer elemento de la sucesión {a1, a2, . . . } tal que
an1 ∈ B. Llamamos an2 al primer elemento que sigue a an1 en la sucesión
anterior tal que an2 ∈ B. Entonces B = {an1 , an2 , . . . }. Si el conjunto de
enteros positivos {n1, n2, . . . } es acotado, B es finito. En caso contrario, B
es numerable.
PROBLEMA 1.26.
Demostrar que
√
2 +
√
3 no puede ser racional.
Solución
Si x =
√
2 +
√
3 =⇒ x2 = 2 + 3 + 2
√
6 =⇒ x2 − 5 = 2
√
6 =⇒ x4 − 10x2 +
1 = 0, y la propia construcción prueba que
√
2 +
√
3 es ráız de la última
ecuación.
Recordamos aqúı el teorema de Ruffini que expresa que toda solución ra-
cional p/q de una ecuación a0xn + a1xn−1 + · · · + an = 0, con ai ∈ Z y
a0 6= 0, an 6= 0, verifica que p divide a an y q divide a a0.
18
En nuestro caso, si p/q es solución racional de la ecuación de coeficientes
enteros x4 − 10x2 + 1 = 0, entonces p y q son divisores de 1. Por tanto, las
únicas soluciones racionales son 1 y -1, que no verifican la ecuación, como
se puede comprobar inmediatamente. Como sabemos que x =
√
2 +
√
3 es
solución, no puede ser racional.
PROBLEMA 1.27.
Sean m y n dos números naturales primos entre śı y
√
m,
√
n
irracionales. Demostrar que también son irracionales i)
√
m ·
√
n
y ii)
√
m +
√
n.
Solución
i) Si fuera
√
m ·
√
n = p/q ∈ Q con p y q primos entre śı, entonces m · n =
p2/q2 =⇒ p2 = m · n · q2, de modo que p2 es múltiplo de q2 y en
consecuencia p es múltiplo de q. Como m.c.d.(p, q) = 1, sólo puede ser
q = 1 y
√
m ·
√
n = p ∈ N.
Descomponemos ahora m,n, p en factores primos:
m = mα11 m
α2
2 . . .m
αr
r , con mi primos;
n = nβ11 n
β2
2 . . . n
βs
s , con ni primos;
p = pγ11 p
γ2
2 . . . p
γt
t , con pi primos;
donde mi 6= nj , ∀i, j, por ser m y n primos entre śı.
Tenemos
√
m ·
√
n =
√
mα11 . . .m
αr
r · nβ11 . . . n
βs
s = p
γ1
1 . . . p
γt
t
=⇒ mα11 . . .m
αr
r · n
β1
1 . . . n
βs
s = p
2γ1
1 . . . p
2γt
t .
Esto implica que cada uno de los factores del primer miembro es
igual a alguno de los factores del segundo miembro, de modo que
mα11 , . . . ,m
αr
r , n
β1
1 , . . . , n
βs
s son cuadrados perfectos, aśı como también
deben serlo m y n. Esto quiere decir que
√
m y
√
n son números na-
turales, lo que contradice la suposición inicial.
19
ii) Si fuera
√
m +
√
n = p/q ∈ Q, con m.c.d.(p, q) = 1, entonces, elevando
al cuadrado,
m+n+2
√
mn =
p2
q2
=⇒
√
mn =
p2/q2 − (m + n)
2
=
p2 − q2(m + n)
2q2
∈ Q
pues p2 − q2(m + n) ∈ Z y 2q2 ∈ Z.
Esto contradice el apartado i); luego
√
m +
√
n es irracional.
C. MÉTODO DE INDUCCIÓN COMPLETA.
El método de inducción completa es un método de demostración de propie-
dades válidas en todo el conjunto N y se basa en el principio de inducción,
que dice lo siguiente:
Si S es un subconjunto de N con las dos condiciones siguientes:
i) 1 ∈ S;
ii) cada vez que k ∈ S, entonces k + 1 ∈ S;
entonces S = N.
De lo anterior se deduce que para que una propiedad P (n) que depende de
n sea cierta ∀n ∈ N, basta comprobar:
i) Que P (1) es cierta, es decir, la propiedad es cierta para n = 1.
ii) Que si suponemos P (k) cierta para cualquier k, entonces P (k + 1) es
cierta.
PROBLEMA 1.28.
Demostrar que 1 + 3 + · · ·+ (2n− 1) = n2, para todo n ∈ N.
20
Solución
Para n = 1 el enunciado es cierto porque 2 · 1− 1 = 12.
Si suponemos que es cierto para n = k, es decir que 1+3+· · ·+(2k−1) = k2,
debemos probar que también lo es para k + 1:
1+3+· · ·+(2k−1)+[2(k+1)−1] = k2+[2(k+1)−1] = k2+2k+1 = (k+1)2,
que corresponde precisamente a la propiedad para k + 1.
PROBLEMA 1.29.
Demostrar las siguientes fórmulas para todo n ∈ N:
a) 12 + 22 + · · ·+ n2 = n(n + 1)(2n + 1)
6
.
b) 13 + 23 + · · ·+ n3 = n
2(n + 1)2
4
.
Solución
a) Para n = 1: 12 =
1(1 + 1)(2 · 1 + 1)
6
.
Suponemos que 12 + 22 + · · ·+ k2 = k(k + 1)(2k + 1)
6
y probaremos
que
12 + 22 + · · ·+ k2 + (k + 1)2 = (k + 1)(k + 1 + 1)[2(k + 1) + 1]
6
:
Por hipótesis,
12 + 22 + . . . + k2 + (k + 1)2 =
k(k + 1)(2k + 1)
6
+ (k + 1)2
=
k(k + 1)(2k + 1) + 6(k + 1)2
6
=
(k + 1)[k(2k + 1) + 6(k + 1)]
6
=
(k + 1)[(k + 2)(2k + 3)]
6
.
21
b)Para n = 1: 13 =
12(1 + 1)2
4
.
Suponemos que 13 + 23 + · · ·+ k3 = k
2(k + 1)2
4
.
Probemos que 13 + 23 + · · ·+ k3 + (k + 1)3 = (k + 1)
2(k + 1 + 1)2
4
:
13 + 23 + · · ·+ k3 + (k + 1)3 = k
2(k + 1)2
4
+ (k + 1)3 =
k2(k + 1)2 + 4(k + 1)3
4
=
(k + 1)2[k2 + 4(k + 1)]
4
=
(k + 1)2(k + 2)2
4
.
PROBLEMA 1.30.
Demostrar que 3 + 2 · 31 + · · ·+ 2 · 3n = 3n+1, ∀n ∈ N.
Solución
Para n = 1, 3 + 2 · 31 = 31+1.
Suponemos que 3 + 2 · 31 + · · ·+ 2 · 3k = 3k+1 para algún k ∈ N.
Probemos que 3 + 2 · 31 + · · ·+ 2 · 3k+1 = 3k+2:
3 + 2 · 31 + · · ·+ 2 · 3k+1 = 3k+1 + 2 · 3k+1 = 3 · 3k+1 = 3k+2.
PROBLEMA 1.31.
Demostrar que para todo n > 1,
1 + 1 · 1! + 2 · 2! + · · ·+ (n− 1) · (n− 1)! = n!
Solución
Para n = 2: 1 + 1 · 1! = 2!
Suponemos que 1 + 1 · 1! + 2 · 2! + · · · + (k − 1) · (k − 1)! = k! y debemos
probar que 1 + 1 · 1! + 2 · 2! + · · ·+ (k− 1) · (k− 1)! + k · k! = (k + 1)!.
22
En efecto: 1 + 1 · 1! + 2 · 2! + · · · + (k − 1) · (k − 1)! + k · k! = k! + k · k! =
k!(1 + k) = (k + 1)!
PROBLEMA 1.32.
Demostrar que para todo n ∈ N:
a)
1
1 · 2
+
1
2 · 3
+ · · ·+ 1
n · (n + 1)
=
n
n + 1
.
b)
1
1 · 3
+
1
3 · 5
+ · · ·+ 1
(2n− 1) · (2n + 1)
=
n
2n + 1
.
Solución
a) Para n = 1:
1
1 · 2
=
1
1 + 1
.
Suponemos que
1
1 · 2
+
1
2 · 3
+ · · ·+ 1
k · (k + 1)
=
k
k + 1
.
Probemos que
1
1 · 2
+
1
2 · 3
+ · · ·+ 1
k · (k + 1)
+
1
(k + 1) · (k + 2)
=
k + 1
k + 1 + 1
:
1
1 · 2
+
1
2 · 3
+ · · ·+ 1
k · (k + 1)
+
1
(k + 1) · (k + 2)
=
k
k + 1
+
1
(k + 1) · (k + 2)
=
k(k + 2) + 1
(k + 1) · (k + 2)
=
k2 + 2k + 1
(k + 1) · (k + 2)
=
(k + 1)2
(k + 1) · (k + 2)
=
k + 1
k + 2
.
b) Para n = 1:
1
1 · 3
=
1
2 · 1 + 1
.
Supongamos que
1
1 · 3
+
1
3 · 5
+ · · ·+ 1
(2k − 1) · (2k + 1)
=
k
2k + 1
,
y probemos que
1
1 · 3
+· · ·+ 1
(2k − 1) · (2k + 1)
+
1
(2(k + 1)− 1) · (2(k + 1) + 1)
=
k + 1
2(k + 1) + 1
.
23
En efecto,
1
1 · 3
+ . . . +
1
(2k − 1) · (2k + 1)
+
1
(2(k + 1)− 1) · (2(k + 1) + 1)
=
k
2k + 1
+
1
(2k + 1) · (2k + 3)
=
k(2k + 3) + 1
(2k + 1) · (2k + 3)
=
2k2 + 3k + 1
(2k + 1) · (2k + 3)
=
(2k + 1)(k + 1)
(2k + 1) · (2k + 3)
=
k + 1
2k + 3
.
PROBLEMA 1.33.
Demostrar que
n∑
h=1
(1 + h + h2) =
n2 + 3n + 5
3
· n, ∀n ∈ N.
Solución
Se verificapara n = 1 pues el primer miembro es
1∑
h=1
(1+h+h2) = 1+1+12 =
3, y el segundo,
12 + 3 · 1 + 5
3
· 1 = 3.
Supongamos que se verifica para n = k, es decir
k∑
h=1
(1 + h + h2) =
k2 + 3k + 5
3
· k
y probemos que también es cierto para n = k:
k+1∑
h=1
(1 + h + h2) =
k∑
h=1
(1 + h + h2) + [1 + (k + 1) + (k + 1)2]
=
k2 + 3k + 5
3
· k + (k + 2) + (k + 1)2
=
k3 + 3k2 + 5k + 3k + 6 + 3k2 + 3 + 6k
3
=
k3 + 6k2 + 14k + 9
3
.
24
Por otra parte,
(k + 1)2 + 3(k + 1) + 5
3
· (k + 1) = (k + 1)
3 + 3(k + 1)2 + 5(k + 1)
3
=
k3 + 3k2 + 3k + 1 + 3k2 + 6k + 3 + 5k + 5
3
=
k3 + 6k2 + 14k + 9
3
.
PROBLEMA 1.34.
Probar que a2n − b2n es divisible por a + b, ∀n ∈ N.
Solución
Para n = 1: a2−b2 es divisible por a+b, porque a2−b2 = (a+b)(a−b).
Suponemos que a2k − b2k es divisible por a + b.
Probemos que a2(k+1) − b2(k+1) es divisible por a + b:
a2(k+1) − b2(k+1) = a2k+2 − b2k+2 = a2a2k − b2b2k
= a2a2k − a2b2k + a2b2k − b2b2k = a2(a2k − b2k) + b2k(a2 − b2).
Por hipótesis, existe C tal que a2k − b2k = C(a + b), de modo que:
a2(k+1) − b2(k+1) = a2(a2k − b2k) + b2k(a2 − b2)
= a2C(a + b) + b2k(a− b)(a + b) = (a + b)[a2C + b2k(a− b)],
lo que prueba la tesis.
PROBLEMA 1.35.
Demostrar que x2n−1 + y2n−1 es divisible por x + y, ∀n ∈ N.
25
Solución
Para n = 1, x2·1−1 + y2·1−1 = x + y, que claramente es divisible por x +
y.
Suponemos que x2k−1 + y2k−1 es divisible por x + y, es decir, existe C tal
que x2k−1 + y2k−1 = C(x + y).
Probemos que x2(k+1)−1 + y2(k+1)−1 es divisible por x + y:
x2(k+1)−1 + y2(k+1)−1 = x2k+1 + y2k+1 = x2x2k−1 + y2y2k−1
= x2x2k−1 − y2x2k−1 + y2x2k−1 + y2y2k−1
= (x2 − y2)x2k−1 + y2(x2k−1 + y2k−1)
= (x− y)(x + y)x2k−1 + y2C(x + y)
= (x + y)[(x− y)x2k−1 + y2C].
PROBLEMA 1.36.
Sean a, b ∈ R fijos. Demostrar que para todo n ∈ N,
an − bn = (a− b)(an−1 + an−2b + · · ·+ abn−2 + bn−1).
Solución
Para n = 1, a1 − b1 = a− b.
Suponemos que ak−bk = (a−b)(ak−1+ak−2b+· · ·+abk−2+bk−1) y debemos
probar que ak+1 − bk+1 = (a− b)(ak + ak−1b + · · ·+ abk−1 + bk):
ak+1 − bk+1 = a · ak − b · bk
= a · ak − a · bk + a · bk − b · bk = a(ak − bk) + bk(a− b)
= a(a− b)(ak−1 + ak−2b + · · ·+ abk−2 + bk−1) + bk(a− b)
= (a− b)(ak + ak−1b + · · ·+ abk−1 + bk).
26
PROBLEMA 1.37.
Demostrar la fórmula del binomio de Newton
(a + b)n =
(
n
0
)
an +
(
n
1
)
an−1b + · · ·+
(
n
n− 1
)
abn−1 +
(
n
n
)
bn.
Solución
Para n = 1, (a + b)1 =
(
1
0
)
a1 +
(
1
1
)
b1 = a + b.
Suponemos que (a + b)k =
(
k
0
)
ak +
(
k
1
)
ak−1b + · · ·+
(
k
k − 1
)
abk−1 +
(
k
k
)
bk
y probemos la fórmula para k+1 -recordemos que
(
m
n
)
+
(
m
n + 1
)
=
(
m + 1
n + 1
)
:
(a + b)k+1 = (a + b)(a + b)k
= (a + b)
[(
k
0
)
ak +
(
k
1
)
ak−1b + · · ·+
(
k
n− 1
)
abk−1 +
(
k
k
)
bk
]
=
(
k
0
)
ak+1 +
[(
k
0
)
+
(
k
1
)]
akb + · · ·+
[(
k
k − 1
)
+
(
k
k
)]
abk +
(
k
k
)
bk+1
=
(
k + 1
0
)
ak+1 +
(
k + 1
1
)
akb + · · ·+
(
k + 1
k
)
abk +
(
k + 1
k + 1
)
bk+1.
PROBLEMA 1.38.
Demostrar que n7 − n es múltiplo de 42, ∀n ∈ N.
Solución
Debido a que 42 = 2 · 3 · 7, debemos probar que n7 − n es múltiplo de 2, 3
y 7.
27
Para n = 1, 17 − 1 = 0 que es evidentemente múltiplo de 42 (en realidad es
múltiplo de cualquier número).
Suponemos que k7 − k es múltiplo de 42, es decir, k7 − k = 42c para algún
c ∈ Z.
Probemos que (k + 1)7 − (k + 1) es divisible por 42. Por una parte,
(k + 1)7 − (k + 1) = (k + 1)[(k + 1)6 − 1]
= (k + 1)[(k + 1)3 − 1][(k + 1)3 + 1]
= (k + 1)[k3 + 3k2 + 3k + 1− 1][k3 + 3k2 + 3k + 1 + 1]
= (k + 1)k[k2 + 3k + 3][k3 + 3k2 + 3k + 2]
= (k + 1)k(k + 2)[k2 + 3k + 3][k2 + k + 1],
lo que da lugar a una expresión múltiplo de 2 y de 3, para cualquier valor
de k, pues en la factorización intervienen tres números naturales consecuti-
vos.
Por otra parte, si aplicamos la fórmula del binomio de Newton,
(k + 1)7 − (k + 1) = k7 + 7k6 + 21k5 + 35k4 + 35k3 + 21k2 + 7k + 1− k − 1
= (k7 − k) + 7(k6 + 3k5 + 5k4 + 5k3 + 3k2 + k)
= 42c + 7d = 7(6c + d),
y resulta una expresión múltiplo de 7. De los dos desarrollos se obtiene el
resultado deseado.
PROBLEMA 1.39.
Demostrar que 32n+2 + 26n+1 es múltiplo de 11, ∀n ∈ N.
Solución
Para n = 1, 32·1+2 + 26·1+1 = 81 + 128 = 19 · 11.
Suponemos que 32k+2+26k+1 es múltiplo de 11, es decir, 32k+2+26k+1 = 11c
para algún c ∈ Z.
28
Probemos que 32(k+1)+2 + 26(k+1)+1 es múltiplo de 11:
32(k+1)+2 + 26(k+1)+1 = 32k+4 + 26k+7 = 9 · 32k+2 + 26 · 26k+1
= 9 · 32k+2 + 64 · 26k+1 = 9(32k+2 + 26k+1) + 55 · 26k+1
= 9 · 11 · c + 5 · 11 · 26k+1 = 11(9 · c + 5 · 26k+1)
como se queŕıa probar.
PROBLEMA 1.40.
Probar que 22n + 15n− 1 es múltiplo de 9, ∀n ∈ N.
Solución
Para n = 1 se verifica porque 22·1 + 15 · 1− 1 = 18 = 2 · 9.
Suponemos que 22k + 15k − 1 es múltiplo de 9, es decir, 22n + 15n− 1 = 9c
para algún c ∈ Z.
Probemos que 22(k+1) + 15(k + 1)− 1 es múltiplo de 9:
22(k+1) + 15(k + 1)− 1 = 4 · 22k + 15k + 14
= 4(22k + 15k − 1)− 45k + 18 = 4 · 9c− 9 · 5k + 9 · 2,
que es múltiplo de 9.
PROBLEMA 1.41.
Sean x1, x2, x3 tres números naturales consecutivos. Demostrar que
x21 + x
2
2 + x
2
3 no es múltiplo de 3 pero x
3
1 + x
3
2 + x
3
3 es múltiplo de 9.
29
Solución
Si escribimos x1, x2, x3 como (n− 1), n, (n + 1), tenemos
(n − 1)2 + n2 + (n + 1)2 = 3n2 + 2 que para n = 2 da como resultado 14,
que no es múltiplo de 3.
Ahora bien, probaremos por inducción que (n−1)3+n3+(n+1)3 = 3n3+6n
es múltiplo de 9, para todo n ≥ 2.
Para n = 2, 3 · 23 + 6 · 2 = 36 = 9 · 4.
Suponemos que 3k3 +6k es múltiplo de 9, es decir, 3k3 +6k = 9c para algún
c ∈ Z, y debemos probar que 3(k + 1)3 + 6(k + 1) es múltiplo de 9:
3(k+1)3+6(k+1) = 3k3+9k2+9k+3+6k+6 = 3k3+6k+9(k2+k+1) = 9c+9d,
que es evidentemente múltiplo de 9.
PROBLEMA 1.42.
Demostrar que el producto de n factores, cada uno de los cuales
es suma de los cuadrados de dos números enteros, se puede expre-
sar como la suma de los cuadrados de otros dos números enteros.
Solución
Debemos demostrar que (a21 +b
2
1)(a
2
2 +b
2
2) . . . (a
2
n +b
2
n) = A
2 +B2, para todo
n ∈ N. Para n = 1 es evidente porque a21 + b21 = A2 + B2 eligiendo A = a1 y
B = b1.
Suponemos que (a21+b
2
1)(a
2
2+b
2
2) . . . (a
2
k+b
2
k) = A
2+B2 y probemos que
(a21 + b
2
1)(a
2
2 + b
2
2) . . . (a
2
k+1 + b
2
k+1) = A
2
1 + B
2
1 :
(a21 + b
2
1)(a
2
2 + b
2
2) . . . (a
2
k+1 + b
2
k+1) = (A
2 + B2)(a2k+1 + b
2
k+1)
= A2a2k+1 + A
2b2k+1 + B
2a2k+1 + B
2b2k+1
= (Aak+1)2 + (Abk+1)2 + (Bak+1)2 + (Bbk+1)2
= (Aak+1)2 + (Bbk+1)2 − 2Aak+1Bbk+1
+(Bak+1)2 + (Abk+1)2 + 2Aak+1Bbk+1
= (Aak+1 −Bbk+1)2 + (Bak+1 + Abk+1)2
= A21 + B
2
1 .
30
PROBLEMA 1.43.
Demostrar que para todo n ≥ 4 se verifica n! > 2n.
Solución
Para n = 4, 4! = 24 > 24 = 16.
Suponemos que k! > 2k y probaremos que (k + 1)! > 2k+1:
(k + 1)! = (k + 1) · k! > (k + 1) · 2k ≥ 2 · 2k = 2k+1.
PROBLEMA 1.44.
Demostrar la desigualdad de Bernoulli (1 + x)n > 1 + nx para n =
2, 3, . . . , si x > −1, x 6= 0.
Solución
Para n = 2: (1 + x)2 = 1 + 2x + x2 > 1 + 2x, porque x 6= 0.
Supongamos que (1+x)k > 1+kx y probemos que (1+x)k+1 > 1+(k+1)x.
En efecto,
(1 + x)k+1 = (1 + x)(1 + x)k > (1 + x)(1 + kx) = 1 + x + kx + kx2
= 1 + (k + 1)x + kx2 > 1 + (k + 1)x.
Nótese que el resultado no es cierto para n = 1, pero seŕıa cierto para todo
n ∈ N si modificamos el enunciado por (1 + x)n ≥ 1 + nx.
31
D. DESIGUALDADES Y VALOR ABSOLUTO.
Es sabido que la relación ≤ permite ordenar el cuerpo de los números reales.
Además la relación es de orden total, lo que quiere decir que dados dos
números reales cualesquiera x e y, se cumple que x ≤ y o bien y ≤ x.
Por otra parte, será importante a lo largo del curso la resolución de inecua-
ciones con soluciones en R. Para ello son de utilidad las siguientes propieda-
des:
a) x < y, z ∈ R =⇒ x + z < y + z.
b) x < y, z > 0 =⇒ x · z < y · z.
c) x < y, z < 0 =⇒ x · z > y · z. (Observa que cambia el sentido de la
desigualdad).
d) x < y, y < z =⇒ x < z.
e) x < y, u < v =⇒ x + u < y + v.
Todas estas propiedadestienen sus análogas si cambiamos el signo < por
cualquiera de >, ≤ ó ≥.
Otras propiedades de las desigualdades se obtienen a partir del concepto de
valor absoluto. Aśı, dado un número real x, su valor absoluto o módulo, que
representamos por |x|, se define como:
|x| = +
√
x2 = máx{x,−x} =
{
x si x ≥ 0,
−x si x < 0.
Geométricamente representa la distancia del punto x al origen de coordena-
das.
Las propiedades básicas son las siguientes:
a) −|x| ≤ x ≤ |x|.
b) |x + y| ≤ |x|+ |y| (desigualdad triangular).
c) |x| ≤ a ⇐⇒ −a ≤ x ≤ a.
d) |x| ≥ a ⇐⇒ x ≥ a ó x ≤ −a.
PROBLEMA 1.45.
Si a, b ≥ 0, demostrar que a + b
2
≥
√
ab.
32
Solución
Partimos de la desigualdad evidente (
√
a−
√
b)2 ≥ 0 y obtenemos sucesiva-
mente:
a− 2
√
ab + b ≥ 0 =⇒ a + b
2
≥
√
ab.
Observación. Este resultado se puede generalizar al siguiente:
a1 + · · ·+ an
n
≥ n
√
a1 . . . an,
lo que indica que la media aritmética de los números a1, . . . an es mayor o
igual a la media geométrica de los mismos.
PROBLEMA 1.46.
Demostrar que, para cualquier a ∈ R, a ≤ máx{1, an}, siendo n ∈ N
fijo.
Solución
Probaremos la propiedad para los distintos valores de a.
Si a > 1, máx{1, an} = an y es evidente que a ≤ an.
Si −1 ≤ a ≤ 1, máx{1, an} = 1 y a ≤ 1.
Si a < −1 y n es par, máx{1, an} = an y a ≤ an. En cambio, si n es impar,
máx{1, an} = 1 ≥ a.
PROBLEMA 1.47.
Demostrar que si a y b son números reales positivos,
a > b ⇐⇒ a2 > b2 ⇐⇒
√
a >
√
b.
33
Solución
i) a > b =⇒ a2 > b2:
Supongamos que a > b. Como a > 0, a · a > b · a.
Como también b > 0, a · b > b · b. Pero a · b = b · a. Luego, a · a = a2 >
b · a > b · b = b2, es decir, a2 > b2.
ii) a2 > b2 =⇒ a > b:
Si a2 > b2, entonces a2− b2 > 0 =⇒ (a− b)(a + b) > 0. Como a, b > 0,
a + b > 0, por lo que a− b > 0, es decir a > b.
iii) a > b =⇒
√
a >
√
b:
Si, por el contrario, fuera
√
a ≤
√
b, como
√
a,
√
b > 0, tenemos que√
a
√
a ≤
√
a
√
b y
√
a
√
b ≤
√
b
√
b. En definitiva, a = (
√
a)2 ≤√
a
√
b ≤ (
√
b)2 = b, lo que contradice la hipótesis.
iv) La demostración de que
√
a >
√
b =⇒ a > b es completamente análoga
a la del apartado i).
PROBLEMA 1.48.
Demostrar que la suma de cualquier número positivo con su rećıpro-
co nunca es menor que 2.
Solución
Se trata de probar que a + 1/a ≥ 2, ∀a > 0. Inecuaciones equivalentes a la
primera son
a2 + 1
a
≥ 2 ⇐⇒ a2 + 1 ≥ 2a ⇐⇒ a2 − 2a + 1 ≥ 0 ⇐⇒ (a− 1)2 ≥ 0
lo que es siempre cierto por ser el cuadrado de un número real.
34
PROBLEMA 1.49.
Si a > 0, a 6= 1 y n ∈ N, demostrar que an+1 + 1
an+1
> an +
1
an
.
Solución
Como en problemas anteriores vamos escribiendo inecuaciones equivalentes
a la que queremos demostrar hasta llegar a una cuya demostración sea más
asequible. Tenemos entonces:
an+1 +
1
an+1
> an +
1
an
⇐⇒ a
2(n+1) + 1
an+1
>
a2n + 1
an
⇐⇒ a2(n+1) + 1 > a2n+1 + a
⇐⇒ a2n+2 − a2n+1 − a + 1 > 0
⇐⇒ (a2n+1 − 1)(a− 1) > 0.
En esta última inecuación, si a > 1, ambos factores son positivos por lo que
su producto también lo será. En cambio si 0 < a < 1, ambos factores son
negativos por lo que su producto será también positivo. En ambos casos se
prueba lo deseado.
PROBLEMA 1.50.
Demostrar que a2 + b2 + c2 > ab + bc + ca, ∀a, b, c ∈ R excepto para
a = b = c.
Solución
Como a2 + b2 > 2ab, b2 + c2 > 2bc, c2 + a2 > 2ca, sumando miembro a
miembro las tres inecuaciones resulta 2(a2 + b2 + c2) > 2(ab + bc + ca) y
dividiendo por 2 obtenemos lo deseado.
35
Si a = b = c se verifica la igualdad, por lo que en general podemos escribir
a2 + b2 + c2 ≥ ab + bc + ca.
PROBLEMA 1.51.
Si a, b, c, d ∈ R son tales que a2 + b2 = 1 y c2 + d2 = 1, demostrar
que ac + bd ≤ 1.
Solución
Sabiendo que (a−c)2 ≥ 0 y (b−d)2 ≥ 0, resultan las desigualdades a2+c2 ≥
2ac y b2 + d2 ≥ 2bd. Sumando ambas, tenemos que a2 + c2 + b2 + d2 ≥
2(ac + bd), es decir 2 ≥ 2(ac + bd) o bien 1 ≥ ac + bd.
PROBLEMA 1.52.
Demostrar que x3 + y3 > x2y + y2x si x, y ∈ R+ (x 6= y).
Solución
La inecuación x3 + y3 > x2y + y2x es equivalente a (x + y)(x2 − xy + y2) >
xy(x + y). Si dividimos por x + y, que es positivo, tenemos que también es
equivalente a x2− xy + y2 > xy, o bien x2− 2xy + y2 > 0 lo cual es siempre
cierto porque x2 − 2xy + y2 = (x− y)2 y x 6= y.
PROBLEMA 1.53.
Demostrar que an +bn > an−1b+abn−1 siempre que a, b ∈ R+ (a 6= b)
y n > 1.
36
Solución
Desigualdades equivalentes a la que queremos probar son
an + bn > an−1b + abn−1 ⇐⇒ an − an−1b− (abn−1 − bn) > 0
⇐⇒ an−1(a− b)− bn−1(a− b) > 0
⇐⇒ (an−1 − bn−1)(a− b) > 0.
Esta última inecuación es siempre cierta porque, si a > b, ambos factores
son positivos y, si a < b, ambos factores son negativos.
PROBLEMA 1.54.
Demostrar:
a) x2 − y2 > x− y si x + y > 1 y x > y.
b) x2 − y2 < x− y si x + y > 1 y x < y.
Solución
a) Si x > y, entonces x− y > 0. De la desigualdad x + y > 1 obtenemos
(x + y)(x− y) > (x− y), es decir x2 − y2 > x− y.
b) Como x < y, x−y < 0. Multiplicando ambos miembros de la desigualdad
x + y > 1 por x− y se invierte el sentido de la desigualdad y se tiene:
(x + y)(x− y) < (x− y) ⇐⇒ x2 − y2 < x− y.
PROBLEMA 1.55.
Sabiendo que la media aritmética de dos números a y b es
a + b
2
,
la media geométrica es
√
ab y la media armónica es
2ab
a + b
, probar
que
a + b
2
>
√
ab >
2ab
a + b
, si a y b son positivos y distintos.
37
Solución
i) La siguiente cadena de equivalencias prueba que (a + b)/2 >
√
ab (otro
método se realizó en el problema 1.45):
a + b
2
>
√
ab ⇐⇒ (a + b)2 > (2
√
ab)2 ⇐⇒ a2 + b2 + 2ab > 4ab
⇐⇒ a2 − 2ab + b2 > 0 ⇐⇒ (a− b)2 > 0.
ii) Veamos ahora por el mismo método que
√
ab >
2ab
a + b
:
√
ab >
2ab
a + b
⇐⇒ ab > (2ab)
2
(a + b)2
⇐⇒ (a + b)2 > 4ab ⇐⇒ (a− b)2 > 0.
PROBLEMA 1.56.
Demostrar que si a, b, c, d son números reales positivos tales que
a
b
>
c
d
, entonces
a + c
b + d
>
c
d
.
Solución
Sumando a ambos miembros de la desigualdad la misma cantidad c/b, tene-
mos:
a
b
>
c
d
=⇒ a
b
+
c
b
>
c
d
+
c
b
=⇒ a + c
b
>
c(b + d)
bd
=⇒ a + c
b + d
>
c
d
.
PROBLEMA 1.57.
Demostrar que
1
x
+
1
y
>
2
x + y
si x, y ∈ R+ (x 6= y).
38
Solución
Escribamos expresiones equivalentes a la inecuación a probar:
1
x
+
1
y
>
2
x + y
⇐⇒ x + y
xy
>
2
x + y
⇐⇒ (x + y)2 > 2xy
⇐⇒ x2 + 2xy + y2 > 2xy ⇐⇒ x2 + y2 > 0,
lo cual es evidentemente cierto, pues x 6= y.
PROBLEMA 1.58.
Demostrar que
a2 + b2
2
≥ ab, ∀a, b ∈ R. ¿En qué caso se verifica la
igualdad?
Solución
Si partimos de la desigualdad a probar, mediante expresiones equivalentes
obtenemos:
a2 + b2
2
≥ ab ⇐⇒ a2 + b2 ≥ 2ab ⇐⇒ a2 + b2 − 2ab ≥ 0 ⇐⇒ (a− b)2 ≥ 0,
que es siempre cierto.
La igualdad se verifica cuando a = b.
PROBLEMA 1.59.
Si a1, a2, . . . , an y b1, b2, . . . , bn son números reales cualesquiera, pro-
bar la desigualdad de Schwarz:
(a1b1 + a2b2 + · · ·+ anbn)2 ≤ (a21 + a22 + · · ·+ a2n)(b21 + b22 + · · ·+ b2n).
39
Solución
Si x ∈ R, se tiene
(a1x + b1)2 + · · ·+ (anx + bn)2 ≥ 0.
Desarrollando y agrupando términos, llegamos a:
A2x2 + 2Cx + B2 ≥ 0,
con A2 = a21 + · · ·+ a2n, B2 = b21 + · · ·+ b2n, C = a1b1 + · · ·+ anbn.
La inecuación anterior se puede escribir, completando cuadrados, como (Ax+
C/A)2 + B2−C2/A2 ≥ 0. Si elegimos x = −C/A2, resulta B2−C2/A2 ≥ 0,
es decir, A2B2 ≥ C2, que es la desigualdad buscada.
PROBLEMA 1.60.
Resolver las siguientes inecuaciones:
a) |x− 3| ≤ 1.
b) |x2 − 1| < 1/2.
c) (x + 1)(x− 1)(x− 2) > 0.
Solución
a) |x− 3| ≤ 1 ⇐⇒ −1 ≤ x− 3 ≤ 1 ⇐⇒ 2 ≤ x ≤ 4 ⇐⇒ x ∈ [2, 4].
b)
|x2 − 1| < 1/2 ⇐⇒ −1/2 < x2 − 1 < 1/2 ⇐⇒ 1/2 < x2 < 3/2
⇐⇒ 1/2 < |x|2 < 3/2 ⇐⇒ 1/
√
2 < |x| <
√
3/
√
2
⇐⇒ 1/
√
2 < x <
√
3/
√
2 (para x ≥ 0)
ó −
√
3/
√
2 < x < −1/
√
2 (para x < 0)
⇐⇒ x ∈
(
−
√
3√
2
,− 1√
2
)
∪
(
1√
2
,
√
3√
2
)
.
40
c) Como (x + 1)(x− 1)(x− 2) = 0 cuando x = −1, x = 1, x = 2, podemos
escribir la siguiente tabla de signos:
x < −1 −1 < x < 1 1 < x < 2 x > 2
x + 1 – + + +
x− 1 – – + +
x− 2 – – – +
(x + 1)(x− 1)(x− 2) – + – +
Entonces (x + 1)(x− 1)(x− 2) > 0 cuando −1 < x < 1 ó x > 2.
PROBLEMA 1.61.
Hallarel conjunto de los valores de x para los que se verifica:
a)
1
x
+
3
2x
≥ 5.
b) x(x + 2) ≤ 24.
c) |x + 2| < |x− 5|.
d)
x
x + 2
>
x + 3
3x + 1
.
Solución
a)
1
x
+
3
2x
≥ 5 ⇐⇒ 5x
2x2
≥ 5 ⇐⇒ 5x ≥ 10x2 ⇐⇒ 2x2 − x ≤ 0 ⇐⇒ x(2x− 1) ≤ 0.
Hacemos la tabla de signos de los factores, teniendo en cuenta que
x(2x− 1) = 0 ⇐⇒ x = 0 ó x = 1/2:
x < 0 0 < x < 1/2 x > 1/2
x – + +
2x− 1 – – +
x(2x− 1) + – +
La solución de la inecuación es pues el intervalo (0, 1/2] (téngase en
cuenta que para x = 0 no tiene sentido la inecuación).
41
b) x(x + 2) ≤ 24 ⇐⇒ x2 + 2x− 24 ≤ 0 ⇐⇒ (x− 4)(x + 6) ≤ 0.
Como el primer miembro de la inecuación se anula en los puntos x = 4
y x = −6, la correspondiente tabla de signos es:
x < −6 −6 < x < 4 x > 4
x + 6 – + +
x− 4 – – +
(x + 6)(x− 4) + – +
La solución de la inecuación es el intervalo cerrado [−6, 4].
c) Para resolver la inecuación |x+2| < |x−5| debemos eliminar los valores
absolutos, sabiendo que |x + 2| =
{
x + 2 si x + 2 ≥ 0
−x− 2 si x + 2 < 0
y |x− 5| ={
x− 5 si x− 5 ≥ 0
−x + 5 si x− 5 < 0
. Por ello descomponemos la recta real en los
siguientes intervalos:
x < −2 −2 ≤ x < 5 5 ≤ x
|x + 2| −x− 2 x + 2 x + 2
|x− 5| −x + 5 −x + 5 x− 5
y resolvemos las inecuaciones que resultan en cada intervalo.
Si x < −2: −x− 2 < −x + 5 ⇐⇒ −2 < 5 lo cual es siempre cierto. El
intervalo (−∞,−2) es solución de la inecuación.
Si −2 ≤ x < 5: x + 2 < −x + 5 ⇐⇒ 2x < 3 ⇐⇒ x < 3/2. La solución
en este caso es [−2, 3/2).
Si 5 ≤ x: x + 2 < x− 5 ⇐⇒ 2 < −5 lo cual es siempre falso.
Uniendo las soluciones parciales tenemos que la solución completa de
la inecuación es el intervalo (−∞, 3/2).
d)
x
x + 2
>
x + 3
3x + 1
⇐⇒ x
x + 2
− x + 3
3x + 1
> 0 ⇐⇒ x(3x + 1)− (x + 3)(x + 2)
(x + 2)(3x + 1)
> 0
⇐⇒ 2x
2 − 4x− 6
(x + 2)(3x + 1)
> 0 ⇐⇒ 2(x− 3)(x + 1)
(x + 2)(3x + 1)
> 0.
Los puntos donde puede cambiar de signo la expresión son los que
anulan numerador o denominador, es decir x = −2,−1,−1/3, 3. La
42
tabla de signos queda de la forma:
x < −2 −2 < x < −1 −1 < x < −13
−1
3 < x < 3 x > 3
x + 2 – + + + +
x + 1 – – + + +
3x + 1 – – – + +
x− 3 – – – – +
2(x−3)(x+1)
(x+2)(3x+1) + – + – +
La tabla indica que la solución de la inecuación es (−∞,−2)∪(−1,−1/3)∪
(3,∞).
43
E. EJERCICIOS PROPUESTOS.
1.- Sea S un subconjunto no vaćıo de R y denotamos por i a una cota
inferior de S. Demostrar que
i = ı́nf S ⇐⇒ ∀ε > 0,∃b ∈ S : i ≤ b < i + ε.
Sugerencia: Ver problema 1.6.
2.- Probar, por el método de inducción completa, que 34n+2 + 52n+1
es múltiplo de 14.
Sugerencia: Ver problema 1.39.
3.- Demostrar que xn − yn es divisible por x− y, ∀n ∈ N.
Sugerencia: Ver problema 1.36.
4.- Probar las siguientes propiedades de los números reales:
(a) |x− y| ≤ |x|+ |y|.
(b) |x| − |y| ≤ |x− y|.
Sugerencia: Elevar al cuadrado los dos miembros de la desigualdad.
(c)
∣∣|x| − |y|∣∣ ≤ |x− y|.
5.- Resolver las siguientes desigualdades:
(a)
1
x
+
1
1− x
> 0.
Resp.: x ∈ (0, 1).
(b) 5− x2 < −2.
Resp.: x ∈ (−∞,−
√
7) ∪ (
√
7,∞).
(c) x2 + x + 1 ≤ 0.
Resp.: ∅.
(d) |1 + 2x| ≤ 1.
Resp.: x ∈ [−1, 0].
(e) |x + 2| ≥ 5.
44
Resp.: x ∈ (−∞,−7] ∪ [3,∞).
(f) |x− 5| < |x + 1|.
Resp.: x > 2.
(g) x < x2 − 12 < 4x.
Resp.: x ∈ (4, 6).
(h)
∣∣∣x+1x−1 ∣∣∣ ≤ 1.
Resp.: x ≤ 0.
6.- Probar que máx(x, y) =
x + y + |y − x|
2
y mı́n(x, y) =
x + y − |y − x|
2
,
∀x, y ∈ R.
Sugerencia: Tener en cuenta que, cuando y ≥ x, |y − x| = y − x y
máx(x, y) = y.
7.- Encontrar el error en los siguientes razonamientos:
(a) Para resolver la desigualdad
x + 1
x− 1
≥ 1, procedemos aśı:
x + 1 ≥ x− 1, de donde 1 ≥ −1, lo que indica que la desigualdad
inicial es válida para todos los números reales. En particular lo
será para x = −1, lo que conduce a
−1 + 1
−1− 1
≥ 1,
es decir, 0 ≥ 1.
(b) De la desigualdad evidente 8 < 16, deducimos la siguiente
cadena de desigualdades:
1
8
>
1
16
⇐⇒ (1/2)3 > (1/2)4
⇐⇒ 3 log(1/2) > 4 log(1/2) ⇐⇒ 3 > 4.
45

Continuar navegando

Materiales relacionados

291 pag.
19 pag.
Números Reales

UFRJ

User badge image

Neisser Arturo Soto Fernández

215 pag.